Difference between revisions of "2021 Mock AMC 8 Problems"

(Problem 1)
(Problem 1)
Line 7: Line 7:
 
\qquad\mathrm{(D)}\ 7
 
\qquad\mathrm{(D)}\ 7
 
\qquad\mathrm{(E)}\ 25</math>
 
\qquad\mathrm{(E)}\ 25</math>
 +
 +
==Problem 2==

Revision as of 15:51, 16 October 2021

Problem 1

What is the value of $1-2+3-4+5-6+7-8+9$?

$\mathrm{(A)}\ 4 \qquad\mathrm{(B)}\ 5 \qquad\mathrm{(C)}\ 6  \qquad\mathrm{(D)}\ 7 \qquad\mathrm{(E)}\ 25$

Problem 2